Difference between revisions of "1980 AHSME Problems/Problem 28"
m (→Solution) |
J314andrews (talk | contribs) |
||
(4 intermediate revisions by 2 users not shown) | |||
Line 9: | Line 9: | ||
\text{(E)} \ 65 </math> | \text{(E)} \ 65 </math> | ||
− | == Solution == | + | ==Solution 1== |
+ | The roots of <math>x^2+x+1</math> are <math>-\frac{1}{2} + \frac{\sqrt{3}}{2}i = \cos\frac{2}{3}\pi + i \sin \frac{2}{3}\pi</math> and <math>-\frac{1}{2} - \frac{\sqrt{3}}{2}i = \cos \frac{4}{3}\pi + i \sin\frac{4}{3}\pi</math>, which are the primitive third roots of unity. | ||
+ | |||
+ | Let <math>w = -\frac{1}{2} + \frac{\sqrt{3}}{2}i = \cos\frac{2}{3}\pi + i \sin \frac{2}{3}\pi</math>. Note that <math>-\frac{1}{2} - \frac{\sqrt{3}}{2}i = \cos \frac{4}{3}\pi + i \sin\frac{4}{3}\pi = w^2 = \bar{w}</math>. So by the conjugate root theorem, <math>x^{2n} + 1 + (1+x)^{2n}</math> is divisible by <math>x^2+x+1</math> if and only if <math>w</math> is a root of <math>x^{2n} + 1 + (1+x)^{2n}</math>. | ||
+ | |||
+ | Also, <math>w^{2n} + 1 + (w+1)^{2n} = w^{2n} + 1 + (-w^2)^{2n} = w^{2n} + 1 + w^{4n} = w^{2n} + 1 + w^n = w^{2n} + w^n + 1</math>. | ||
+ | |||
+ | If <math>n \equiv 0\ (\textrm{mod}\ 3)</math>, <math>w^{2n} + w^n + 1 \equiv 1 + 1 + 1 = 3</math>. | ||
+ | |||
+ | If <math>n \equiv 1\ (\textrm{mod}\ 3)</math>, <math>w^{2n} + w^n + 1 \equiv w^2 + w + 1 = 0</math>. | ||
+ | |||
+ | If <math>n \equiv 2\ (\textrm{mod}\ 3)</math>, <math>w^{2n} + w^n + 1 \equiv w + w^2 + 1 = 0</math>. | ||
+ | |||
+ | Therefore, <math>x^{2n} + 1 + (1+x)^{2n}</math> is not divisible by <math>x^2 + x + 1</math> if and only if <math>n</math> is divisible by <math>3</math>. Of the answer choices, only <math>\boxed{(\textbf{C})\ 21}</math> is divisible by <math>3</math>. | ||
+ | |||
+ | == Solution 2== | ||
Let <math>h(x)=x^2+x+1</math>. | Let <math>h(x)=x^2+x+1</math>. | ||
− | Then | + | Then <math>(x+1)^{2n} = (x^2+2x+1)^n = (x+h(x))^n = \sum_{k=0}^{n} \binom{n}{k} x^k (h(x))^{n-k} </math> <math>= \sum_{k=0}^{n-1} \binom{n}{k} x^k (h(x))^{n-k} + x^n = h(x) \left(\sum_{k=0}^{n-1} \binom{n}{k} x^k (h(x))^{n-k-1}\right) + x^n</math>. |
− | < | + | |
− | + | Let <math>g(x) = \sum_{k=0}^{n-1} \binom{n}{k} x^k (h(x))^{n-k-1}</math>. Then <math>x^{2n} + 1 + (x+1)^{2n} = x^{2n} + 1 + g(x)h(x) + x^n</math>, so <math>x^{2n} + 1 + (x+1)^{2n}</math> is divisible by <math>x^2 + x + 1</math> if and only if <math>x^{2n} + x^n + 1</math> is divisible by <math>x^2 + x + 1</math>. | |
+ | |||
+ | Let <math>k</math> be any non-negative integer, and let <math>r \in \{0, 1, 2\}</math>. Then <math>x^{3k+r} - x^r = x^r(x^{3k} - 1) = x^r(x^3 - 1)\left(\sum_{j=0}^{k - 1} x^{3j}\right) </math> | ||
+ | <math>= x^r(x-1)(x^2 + x + 1)\left(\sum_{j=0}^{k - 1} x^{3j}\right)</math>. | ||
+ | |||
+ | Therefore, <math>x^{3k+r} - x^r</math> is divisible by <math>x^2 + x + 1</math>. | ||
+ | |||
+ | If <math>n</math> is a nonnegative integer, then <math>n</math> can be written as <math>3k + r</math>, where nonnegative integer <math>k</math> and <math>r \in \{0, 1, 2\}</math> are the quotient and remainder when <math>n</math> is divided by <math>3</math>. | ||
+ | |||
+ | If <math>r = 2</math>, then <math>x^{2n} + x^n + 1 = x^{6k+4} + x^{3k+2} + 1 = x^{3(2k+1)+1} + x^{3k+2} + 1 = (x^{3(2k+1)+1} - x) + (x^{3k+2} - x^2) + x^2 + x + 1</math>. Since <math>x^{3(2k+1)+1} - x</math> and <math>(x^{3k+2} - x^2)</math> are divisible by <math>x^2 + x + 1</math>, <math>x^{2n} + x^n + 1</math> is divisible by <math>x^2 + x + 1</math>. | ||
+ | |||
+ | If <math>r = 1</math>, then <math>x^{2n} + x^n + 1 = x^{6k+2} + x^{3k+1} + 1 = x^{3(2k)+2} + x^{3k+1} + 1 = (x^{3(2k)+2} - x^2) + (x^{3k+1} - x) + x^2 + x + 1</math>. Since <math>x^{3(2k)+2} - x^2</math> and <math>(x^{3k+1} - x)</math> are divisible by <math>x^2 + x + 1</math>, <math>x^{2n} + x^n + 1</math> is divisible by <math>x^2 + x + 1</math>. | ||
+ | |||
+ | If <math>r = 0</math>, then <math>x^{2n} + x^n + 1 = x^{6k} + x^{3k} + 1 = (x^{6k} - 1) + (x^{3k} - 1) + 3</math>. Since <math>x^{6k} - 1</math> and <math>x^{3k} - 1</math> are divisible by <math>x^2 + x + 1</math>, <math>x^{2n} + x^n + 1</math> is not divisible by <math>x^2 + x + 1</math>. | ||
+ | |||
+ | So <math>x^{2n} + 1 + (1+x)^{2n}</math> is not divisible by <math>x^2 + x + 1</math> if and only if <math>r = 0</math>, that is, <math>n</math> is divisible by <math>3</math>. The only answer choice that is divisible by <math>3</math> is <math>\boxed{(\textbf{C})\ 21}</math>. | ||
− | + | -Wei, edited by j314andrews | |
− | |||
− | |||
− | |||
− | < | + | ==Solution 3== |
− | + | We start by noting that <cmath>x + 1 \equiv -x^2 \mod (x^2+x+1).</cmath> | |
− | + | Let <math>n = 3k+r</math>, where <math>r \in \{ 0,1,2 \}</math>. | |
− | + | Thus we have <cmath>x^{4n} + x^{2n} + 1 \equiv x^{4r} + x^{2r} + 1 \mod (x^3 -1).</cmath> | |
− | |||
− | + | When <math>r = 0</math>, <cmath>x^{4n} + x^{2n} + 1 \equiv 3 \mod (x^3 -1).</cmath> | |
− | + | When <math>r = 1</math>, <cmath>x^{4n} + x^{2n} + 1 \equiv x^2 + x + 1 \mod (x^3 -1),</cmath> which will be divisible by <math>x^2+x+1</math>. | |
− | + | When <math>r = 2</math>, <cmath>x^{4n} + x^{2n} + 1 \equiv x^2 + x + 1 \mod (x^3 -1),</cmath> which will also be divisible by <math>x^2+x+1</math>. | |
− | + | Thus <math>r \ne 0</math>, so <math>n</math> cannot be divisible by <math>3</math>, and the answer is <math>\textrm{(C)}</math>. | |
== See also == | == See also == |
Latest revision as of 20:03, 17 August 2025
Problem
The polynomial is not divisible by
if
equals
Solution 1
The roots of are
and
, which are the primitive third roots of unity.
Let . Note that
. So by the conjugate root theorem,
is divisible by
if and only if
is a root of
.
Also, .
If ,
.
If ,
.
If ,
.
Therefore, is not divisible by
if and only if
is divisible by
. Of the answer choices, only
is divisible by
.
Solution 2
Let .
Then
.
Let . Then
, so
is divisible by
if and only if
is divisible by
.
Let be any non-negative integer, and let
. Then
.
Therefore, is divisible by
.
If is a nonnegative integer, then
can be written as
, where nonnegative integer
and
are the quotient and remainder when
is divided by
.
If , then
. Since
and
are divisible by
,
is divisible by
.
If , then
. Since
and
are divisible by
,
is divisible by
.
If , then
. Since
and
are divisible by
,
is not divisible by
.
So is not divisible by
if and only if
, that is,
is divisible by
. The only answer choice that is divisible by
is
.
-Wei, edited by j314andrews
Solution 3
We start by noting that
Let
, where
.
Thus we have
When ,
When
,
which will be divisible by
.
When ,
which will also be divisible by
.
Thus , so
cannot be divisible by
, and the answer is
.
See also
1980 AHSME (Problems • Answer Key • Resources) | ||
Preceded by Problem 27 |
Followed by Problem 29 | |
1 • 2 • 3 • 4 • 5 • 6 • 7 • 8 • 9 • 10 • 11 • 12 • 13 • 14 • 15 • 16 • 17 • 18 • 19 • 20 • 21 • 22 • 23 • 24 • 25 • 26 • 27 • 28 • 29 • 30 | ||
All AHSME Problems and Solutions |
These problems are copyrighted © by the Mathematical Association of America, as part of the American Mathematics Competitions.